MPRE

¡Supera tus tareas y exámenes ahora con Quizwiz!

A lawyer may reveal confidential information concerning the representation of a client to the extent the lawyer reasonably believes it necessary to establish a claim or defense on behalf of the lawyer in a controversy between the lawyer and the client, to establish a defense to a criminal charge or civil claim against the lawyer based on conduct in which the client was involved, or to respond to allegations in any proceeding concerning the lawyer's representation of the client. MRPC 1.6(b)(5).

A client disputes a fee owed to his attorney for representation in a criminal defense matter. The attorney is permitted to prove the services rendered in the defense action in order to collect the fee.

A lawyer may not accept payment for representation from someone other than the client, unless: (i) the client gives informed consent; (ii) there is no interference with the lawyer's professional judgment; and (iii) client-lawyer confidentiality is preserved. MRPC 1.8(f).

A common exam question involves a parent paying for representation of a son or daughter. Remember that a competent child, not the parent, is the client whose expressed interests govern.

The Model Rules prohibit fee sharing by lawyers with non-lawyers except when: i) Fees are paid into a lawyer's estate or to one or more specified persons over a reasonable period of time as a death benefit according to an operating agreement; ii) A lawyer who purchases the practice of a deceased, disabled, or disappeared lawyer pays to that lawyer's estate or other representative the agreed-upon purchase price; iii) Fees are shared with law firm personnel via a compensation or retirement plan, even though it is based on a profit-sharing arrangement; and iv) A lawyer shares court-awarded legal fees with a non-profit organization that employed, retained, or recommended employment of the lawyer in the matter.

A law firm is organized as a professional corporation with five lawyer-shareholders. It is improper for a widow, whose husband was a lawyer-shareholder in the law firm until his death three years ago, to continue to hold her husband's shares in the law firm until their ten-year old child completes a college education. By contrast, the law firm can properly pay fees to the husband's estate or to the widow over a reasonable period of time as a death benefit according to an operating agreement, or could pay an agreed-upon purchase price to the husband's estate for his shares.

A lawyer is not permitted to practice with or in the form of a professional corporation or association authorized to practice law for a profit, if: i) A non-lawyer owns any interest therein (except that a fiduciary representative of the estate of a lawyer may hold the stock or interest of the lawyer for a reasonable time during administration); ii) A non-lawyer is a corporate director or officer thereof or occupies a position of similar responsibility in any form of association other than a corporation; or iii) A non-lawyer has the right to direct or control the professional judgment of a lawyer.

A law firm is organized as a professional corporation with five lawyer-shareholders. It would be improper for the office manager, who is not a member of the bar, to serve as the executive vice president of the law firm.

Due to the potential conflict between the interests of a client and the interest of a lawyer in testifying truthfully, a lawyer generally is disqualified from representing a client if the lawyer is likely to be a necessary witness in the action (i.e., the advocate-witness rule).

A lawyer is not permitted to act as an advocate at a trial in which the lawyer is likely to be a necessary witness unless: i) The testimony relates to an uncontested issue; ii) The testimony relates to the nature and value of legal services rendered in the case; or iii) Disqualification of the lawyer would work substantial hardship on the client. MRPC 3.7(a).

Lawyers in the same law firm, by agreement, may legitimately share or split fees earned by any of them. Fees may even be shared with a lawyer who has retired from the firm. MRPC 1.5, cmt. 8.

A lawyer is not permitted to form a partnership with a non-lawyer if any of the activities of the partnership consist of the practice of law.

A lawyer who knowingly prepared a material false document for submission by a client may be subject to criminal liability. Restatement (Third) of the Law Governing Lawyers § 8.

A lawyer may be liable to a client for professional negligence if the lawyer fails to diligently pursue a matter. MRPC 1.3; Restatement (Third) of the Law Governing Lawyers § 16 cmt. a.

Property is generally an acceptable form of payment, but the payment is subject to requirements of MRPC 1.8(a) (see V.D.1. Business transactions with client, infra). Payment in the form of a proprietary interest in the cause of action or subject matter of the litigation must also comply with MRPC 1.8(i) (see V.C.1. Proprietary interest in causes of action, infra).

A lawyer may not make an agreement with a client the terms of which might induce the lawyer improperly to curtail services for the client or perform them in a way contrary to the client's interest. MRPC 1.5, cmt. 5. Thus, a lawyer may not enter into an agreement under which services are to be provided only up to a stated amount when it is foreseeable that more extensive services will likely be required, unless the situation is adequately explained to the client. Otherwise, the client might have to bargain for further assistance in the midst of a proceeding or transaction. However, it is proper to define the extent of services in light of the client's ability to pay.

A lawyer may not enter into an agreement with a client prospectively limiting malpractice liability to the client, unless the client is represented by another independent lawyer in making the agreement. MRPC 1.8(h)(1). See V.D.4. Promise to limit malpractice recovery, supra.

A lawyer may not settle a claim or potential claim for malpractice liability with an unrepresented client or former client unless that person is advised in writing of the desirability of seeking independent legal counsel with regard to the settlement and is given a reasonable opportunity to seek such advice. MRPC 1.8(h)(2).

A lawyer cannot justify a lack of diligence based on illness, subordinates, personal animosity, or the inability to balance other work. A lawyer must control her workload to ensure that all matters are handled competently. MRPC 1.3, cmt. 2.

A lawyer must act with reasonable promptness in representing a client. A client's interests can often be adversely affected by the passage of time or a change of conditions. Moreover, unreasonable delay can cause a client anxiety and undermine confidence in the trustworthiness of the lawyer. A lawyer's duty to act with reasonable promptness, however, does not prohibit the lawyer from agreeing to a reasonable request for a postponement that will not prejudice the lawyer's client. MRPC 1.3, cmt. 3. Misleading a client about the status of the lawyer's progress is its own violation separate from a diligence violation. MRPC 1.4, cmt. 7.

A lawyer lacking the necessary knowledge or experience must: i) Decline or withdraw from representing the client; ii) Become competent without unreasonable delay; or iii) Associate with competent counsel.

A lawyer should not accept representation in a matter unless it can be performed competently. MRPC 1.16, cmt. 1. A lawyer can accept representation of a client when the lawyer reasonably believes that she can become competent without unreasonable delay without disclosing her lack of competency to the client. A lawyer who represents a client incompetently is subject to the punishment determined by the disciplinary tribunal, regardless of causation or any financial loss to the client. A client cannot consent to the less than competent representation. Even if the client does "consent" (perhaps because the lawyer's services are offered at a low fee or because of friendship), the lawyer providing incompetent services is subject to discipline.

In general, a lawyer is only subject to discipline for her own conduct and is not subject to discipline for the conduct of a partner, associate, or subordinate merely because both belong to the same law firm. MRPC 5.1., cmt. 7.

A lawyer who is practicing law as a partner in a partnership may have civil liability for the conduct of another lawyer-partner.

A lawyer may reveal information relating to the representation of a client to the extent the lawyer reasonably believes necessary to prevent reasonably certain death or substantial bodily harm. MRPC 1.6(b)(1). A lawyer who reveals such information has behaved in a professionally appropriate manner.

A lawyer who represents a client in a civil action may be subject to disqualification due to a conflict of interest with another current client. MRPC 1.7(a).

A lawyer may not represent anyone in connection with a matter in which the lawyer participated personally and substantially as a judge or other adjudicative officer, law clerk, arbitrator, mediator or other neutral third party, unless all parties to the proceeding give informed consent, confirmed in writing. MRPC 1.12(a). A judge who was a member of a multi-member court, and then returned to private practice, would not be prohibited from representing a client in a matter pending in the court, but in which the former judge did not participate. Similarly, the fact that a former judge exercised administrative responsibility in a court would not prevent him from acting as a lawyer in a matter in which the judge had previously exercised remote or incidental administrative responsibility that did not affect the merits of the case. MRPC 1.12, cmt. 1.

An arbitrator selected as a partisan of a party in a multi-member arbitration panel is not prohibited from subsequently representing such party. MRPC 1.12(d). If a lawyer is disqualified for having participated personally and substantially as a judge or other adjudicative officer, law clerk, arbitrator, mediator or other third-party neutral, no lawyer in a firm with which that lawyer is associated may knowingly undertake or continue representation in the matter unless: (i) the disqualified lawyer is timely screened from any participation in the matter and gets no part of the fee; and (ii) written notice is promptly given to the parties and any appropriate tribunal to enable them to determine if the ethics rules have been met. MRPC 1.12(c). A lawyer who is participating personally and substantially as a judge or other adjudicative officer, arbitrator, mediator or other third-party neutral may not negotiate for employment with any person who is involved as a party or lawyer for a party in the matter. A lawyer serving as a law clerk may negotiate for employment with a party or lawyer involved in a matter in which the clerk is participating personally and substantially, but only after the lawyer has notified the judge or other adjudicative officer for whom the lawyer works. MRPC 1.12(b).

A subordinate lawyer must conform to the Model Rules even if acting under the direction of a supervising lawyer. MRPC 5.2. The subordinate lawyer is not in violation of the Model Rules, however, if the subordinate lawyer acts in accordance with the supervising lawyer's reasonable resolution of an arguable question of professional duty. MRPC 5.2(b). If an ethical issue can only be reasonably answered in one specific way, it does not constitute an arguable question. MRPC 5.2, cmt. 2. Also, in order to know whether a supervising lawyer's resolution is in fact reasonable, the subordinate lawyer has a responsibility to undertake his own analysis of the law applicable to his conduct.

An associate attorney disagreed with his supervising attorney as to their responsibility to respond to a request for production of documents. The request is ambiguous and after careful consideration of the rules, the associate attorney follows his supervisor's direction not to produce certain documents. A court subsequently finds that the refusal to produce the documents constituted a violation of the Model Rules. The supervising attorney is subject to discipline, but the associate attorney is not.

A person who consults with a lawyer about the possibility of forming a client-lawyer relationship with respect to a matter is a prospective client. MRPC 1.18(a). Whether communications (written, oral, or electronic) constitute a consultation depends on the circumstances.

An attorney is preparing to file a class action suit against a large pharmaceutical company, alleging that an asthma medication causes kidney failure. The attorney runs a television advertisement inviting members of the public who have taken the medication to call him. After seeing the ad, a woman calls the attorney and tells him about her experiences taking the medication. The woman has likely "consulted" with the attorney. Contrast: An attorney advertises his personal injury practice on television, including a statement that he "fights for the rights of victims." After seeing the ad, a mother telephones the attorney, tells him that her son has been taking a particular asthma medication for two years, and asks if it is true that the drug causes kidney failure. It is unlikely that this conversation would be considered a "consultation."

Authority arising from retention does not extend to matters, such as approving a settlement, that are reserved for client decision. When a lawyer purports to enter a settlement binding on the client but lacks authority to do so, the burden of inconvenience resulting if the client repudiates the settlement is properly left with the opposing party, who should know that settlements are normally subject to approval by the client and who has no manifested contrary indication from the client. The opposing party can protect itself by obtaining clarification of the lawyer's authority. Restatement (Third) of the Law Governing Lawyers § 27, cmt. d. See § 2.b.1,

An attorney represents a client in a civil action in which the court has ordered the lawyers from both sides to appear at a pre-trial conference with authority to settle the case or to arrange for a person so authorized to attend. The client was not informed of the court's order and has not authorized the attorney to approve any settlement. The attorney, without disclosing her lack of authority, attends the pre-trial conference and agrees to a settlement with the opposing party. The client is not bound by this settlement and the attorney is subject to disciplinary sanctions, including contempt of court, and may be liable to the opposing party for damages.

The owner of undeveloped real property entered into a contract to sell the property to an investor. The investor's obligation to purchase the property was conditioned on the title to the property being marketable. A lawyer employed by the investor investigated the validity of the seller's title to the property and reported to the investor that there was a cloud on the seller's title, which made the property unmarketable. As a consequence, on the date set for closing, the investor did not tender the purchase price. On that date, due to independent economic events, the fair market value of the property was less than the purchase price. Subsequently, it was uncovered that the lawyer misread a key document and the seller had clear title to the property. Is the lawyer subject to discipline? A. Yes, because the lawyer failed to competently investigate the validity of the seller's title to the property. B. Yes, because the seller suffered an economic loss when the buyer refused to complete the sale. C. No, because due to the drop in the fair market value of the property, the investor did not suffer an economic loss in foregoing the purchase of the property. D. No, because the lawyer's error was not intentional.

Answer choice A is correct. A lawyer is required to provide competent representation to a client. Here, the lawyer failed to provide such representation by negligently conducting the title search. Answer choice B is incorrect because, although the seller presumably suffered an economic loss when the seller eventually sold the property since, at the time of closing, the property's fair market value had dropped below the contract price, it is not necessary that a third party suffer harm for a lawyer to be subject to discipline for violation of a rule. Answer choice C is incorrect because, although the investor did not suffer an economic loss by not completing the sale since the fair market value of the property had dropped below its contract price, it is not necessary that a client suffer harm for a lawyer to be subject to discipline for a violation of a rule. Answer choice D is incorrect because, while the punishment imposed for violation of a rule of professional conduct may be less when the lawyer's violation was not willful, a lawyer is nevertheless subject to discipline for violation of a rule even though the violation was inadvertent rather than intentional.

In an attempt to grow her business, a newly licensed attorney accepted nearly all cases brought to her. After a few months, her workload became so heavy that she did not have sufficient time to devote to the cases, and she subsequently hired a paralegal and a secretary to alleviate some of the work. She also telephoned each of her clients and informed them that her secretary and paralegal would complete much of the legal legwork because she simply did not have time to do so herself. The attorney subsequently missed a major deadline, and a client filed a malpractice suit against her. During discovery, the attorney stated that her paralegal was responsible for tracking deadlines and that her excessive workload prevented her from being able to work full-time on each case herself. Were the attorney's actions with regard to managing her workload proper? A. No, because she had a duty to manage her workload more effectively. B. No, because the attorney did not inform her clients in writing that her staff would take on some of the work. C. Yes, because it was not possible for the attorney to work on all details of each case in light of her workload. D. Yes, because the attorney adequately supervised her staff.

Answer choice A is correct. An attorney cannot justify a lack of diligence based on illness, subordinates, personal animosity, or the inability to balance other work. An attorney must control her workload to ensure that she can handle all matters competently. Here, the attorney took on more cases than she could handle, and she was unable to competently handle her cases, despite the fact that she hired additional staff to compensate for having an unmanageable caseload. Answer choice B is incorrect because an attorney is permitted to employ paraprofessionals, such as secretaries and paralegals, to help the attorney with certain duties, so long as the attorney properly supervises the paraprofessionals. There is no requirement that the attorney inform her clients in writing that she delegated duties to such paraprofessionals. Answer choice C is incorrect because an excessive workload is not a valid excuse to justify a lack of diligence. Answer choice D is incorrect because, although the attorney supervised her staff, she could not allow them to take sole responsibility for all deviations from reasonable diligence.

An attorney represented an incorporated delivery business in a negligence lawsuit stemming from an accident involving a driver employed by the business. The attorney was selected and directed by the president and sole shareholder of the business. After interviewing another employee who witnessed the accident, the attorney engaged in sexual relations with this witness-employee. The attorney did not discuss his professional responsibility obligations regarding sexual relations with the witness-employee. Is the attorney subject to discipline for engaging in sexual relations with the witness-employee? A. No, because the witness-employee did not supervise, direct, or regularly consult with the attorney concerning the delivery business's legal matters. B. No, because the conflict of interest arising from sexual relations with a client does not apply when the client is an entity rather than an individual. C. Yes, because the attorney engaged in a sexual relationship with an employee of the attorney's client. D. Yes, because the attorney did not seek the informed consent of the witness-employee before engaging the sexual relations.

Answer choice A is correct. An attorney is generally prohibited from engaging in sexual relations with a client. When the client is an organization, this prohibition applies to an employee of the client who supervises, directs, or regularly consults with the attorney concerning the organization's legal matters. Since the witness-employee was not such an employee, the prohibition against sexual relations does not apply. Answer choice B is incorrect because the prohibition of engaging in sexual relations with a client may apply to employees of a client that is an organization. Answer choice C is incorrect because while an attorney may be subject to discipline for engaging in a sexual relationship with an employee of a client, a conflict of interest does not arise with respect to all employees, but only those employees who supervise, direct, or regularly consult with the attorney. Answer choice D is incorrect because a client cannot waive this prohibition through informed consent.

A well-known defense attorney met with a criminal defendant regarding representation of the defendant in a highly publicized case. During the meeting, the attorney told the defendant, who was indigent, that she would represent him if he agreed to grant her movie rights regarding the representation. The client agreed to these terms, and the attorney provided him with a written consent form setting forth the terms of the representation and advising him to seek independent counsel. The attorney met with the defendant several days later, at which time he returned a signed copy of the written consent form. He told her that he had not consulted with another attorney. The attorney succeeded in obtaining an acquittal at trial and began shopping a movie based on the case to television studios shortly thereafter. Were the attorney's actions in securing the movie rights based on the case proper? A. No, because the attorney negotiated for movie rights prior to the conclusion of the representation. B. No, because the defendant did not obtain independent legal counsel before signing the consent form. C. Yes, because the defendant signed a written consent form after being advised of his right to seek independent counsel. D. Yes, because the attorney negotiated for movie rights as a replacement for fees.

Answer choice A is correct. An attorney is prohibited from negotiating for literary or media rights relating to representation of a client prior to the conclusion of the representation. Because the attorney negotiated for movie rights before the representation concluded in this case, her actions were improper. Answer choice B is incorrect because the prohibition on making or negotiating an agreement prior to the conclusion of the representation is a blanket prohibition that cannot be waived by the client regardless of whether the client sought or obtained independent legal counsel. Answer choice C is incorrect because an attorney may not negotiate for media rights before the conclusion of representation, even if the client consents. Answer choice D is incorrect because an attorney may not negotiate for literary or media rights before the conclusion of representation, even if such rights serve as the attorney's compensation.

During the trial of an age discrimination lawsuit, both the judge and the attorney referenced the plaintiff's age and political affiliation multiple times in closing argument and jury instructions. The judge stated that "while members of an older generation may be more sensitive to being passed over for promotions, the law does not allow for actual discrimination." He also stated that individuals of the plaintiff's political party might face additional challenges in the particular field, which tends to be composed mostly of members of an opposing political party, and that this fact may have made the plaintiff feel alienated by his peers. Is it proper for the judge to reference the plaintiff's age and political affiliation in the above-referenced manner? A. No, as to both age and political affiliation B. Yes, as to both age and political affiliation C. Yes, as to age only D. Yes, as to political affiliation only

Answer choice A is correct. In performing judicial duties, a judge must not by words or conduct manifest bias or prejudice. Examples of manifestations of bias or prejudice include, but are not limited to: negative stereotyping; attempted humor based upon stereotypes; suggestions of connections between race, ethnicity, or nationality and crime; and irrelevant references to personal characteristics. The requirements of the Code of Judicial Conduct do not preclude judges or attorneys from making legitimate reference to race, sex, gender, religion, national origin, ethnicity, disability, age, sexual orientation, marital status, socioeconomic status, or political affiliation, or similar factors, when they are relevant to an issue in a proceeding. Here, the judge's reference to age reflected negative stereotyping, and was not permitted. Political affiliation was not relevant at all to an age discrimination issue, so the reference was not a legitimate exercise of judicial authority. Answer choice B is incorrect because the judge's comments for both age and political affiliation were improper. Answer choice C is incorrect because the judge's comments about age reflected inappropriate stereotyping. Answer choice D is incorrect because the judge should not have referenced the plaintiff's political affiliation, which was not a relevant issue.

An attorney was retained to represent a client charged with assault. The attorney interviewed several of the client's friends and colleagues in a search of character witnesses. In one such interview, the client's secretary revealed that the client, who was a banker, often skirted legal and ethical lines. After the client was acquitted, the attorney's friend called to congratulate him. The attorney mentioned that it might not be long before the client found himself in legal trouble again, and he told his friend about the conversation with the client's secretary. Was the attorney's action in revealing what he learned from the client's secretary proper? A. No, because the attorney was bound by the duty of confidentiality. B. No, because the conversation with the secretary was protected by attorney-client privilege. C. Yes, because the attorney's representation of the client had concluded. D. Yes, because the conversation was not protected by the attorney-client privilege.

Answer choice A is correct. Under the Model Rules of Professional Conduct, an attorney is prohibited from disclosing information relating to the representation of a client unless the client consents, the disclosure is impliedly authorized, or certain exceptions apply. The ethical duty of confidentiality applies to all information relating to the representation regardless of the source, including third-party communications. Because the attorney learned the information during the course of representation, he was bound by the duty of confidentiality not to disclose it. Answer choice B is incorrect because the attorney-client privilege applies only to communications between an attorney and a client, and only to those communications in which the client expresses a desire for confidentiality. Because this conversation involved a third party, rather than the client, the privilege would not apply. Answer choice C is incorrect because the duty of confidentiality applies to a former client. Answer choice D is incorrect because although attorney-client privilege would not apply, the attorney would be bound by the duty of confidentiality.

A woman had a meeting with an attorney to discuss the attorney's possible representation of her in a divorce. They discussed the facts and circumstances of the divorce, as well as the attorney's fees. At the conclusion of the meeting, the woman told the attorney that she looked forward to working with him. The attorney sent her home with a representation agreement, which he told her to sign and return to him with the discussed retainer. Later that day, the woman's husband, who was a senior manager of a large corporation, asked the attorney to represent him. The attorney realized that he might be able to get other business from the husband and agreed to represent him. When the wife returned the signed representation agreement, the attorney informed her that he was now representing her husband.Is the attorney's representation of the husband proper? A. No, because the wife had discussed the facts and circumstances of her divorce with the attorney. B. No, because the attorney gave the wife a representation agreement to sign. C. Yes, because the wife and the attorney did not enter into an attorney-client relationship. D. Yes, because the wife had not paid a retainer when the attorney agreed to represent the husband.

Answer choice A is correct. While the wife may be able to successfully argue that she entered an attorney-client relationship with the attorney, at the very least, the wife was a prospective client of the attorney. A lawyer may not represent a client with interests materially adverse to those of a prospective client in the same or a substantially related matter if the lawyer received information from the prospective client that could be significantly harmful to that person in the matter. Here, the attorney and the wife discussed the details of the case, which would certainly be harmful to the wife. Accordingly, it was improper for the attorney to accept representation of the husband. Answer choice B is incorrect because, while the representation agreement is certainly evidence of an attorney-client relationship, which would make representation of the husband improper, the agreement alone does not make the representation improper. Additionally, the representation would be improper even if the attorney had not given the wife an agreement. Accordingly, answer choice A is a better answer. Answer choice C is incorrect because, even if no attorney-client relationship existed, an attorney may be prohibited from representing a client whose interests are materially adverse to a prospective client. Here, the wife was a prospective client of the attorney. Because the attorney and the wife discussed material details of the case, which would be harmful to the wife, the attorney's representing the husband is improper, even though no attorney-client relationship existed between the wife and the attorney. Answer choice D is incorrect because the payment of a fee is unnecessary to create an attorney-client relationship.

An attorney was subpoenaed by a grand jury and asked questions regarding the whereabouts of her client. The attorney considered whether the lawyer-client privilege applied to the revelation of this information. Determining that it was more likely than not that the privilege did not apply, the attorney answered the grand jury questions. Is the attorney subject to discipline for disclosing her client's whereabouts? A. Yes, because the attorney failed to assert a claim that the lawyer-client privilege applied. B. Yes, because an attorney may not reveal the whereabouts of her client. C. No, because the attorney determined that it was more likely than not that the lawyer-client privilege did not apply. D. No, because the duty of confidentiality does not apply when an attorney is called as a witness.

Answer choice A is correct. While there is an exception to the duty of confidentiality if the lawyer reasonably believes it necessary to comply with a court order or other law, this exception states that the lawyer should assert all non-frivolous claims that the information sought is protected against disclosure by the lawyer-client privilege or other applicable law. Even though the attorney did not think that the attorney-client privilege applied, asserting the claim on the client's behalf would have been non-frivolous. Answer choice B is incorrect because, while the client's whereabouts is typically information protected by the duty of confidentiality, a lawyer is permitted to reveal such information to the extent that the lawyer reasonably believes it necessary to comply with the law or a court order. Answer choice C is incorrect because the "compliance with other law or court order" exception to the duty of confidentiality does not apply unless the lawyer has first asserted all non-frivolous claims that the information sought is protected against disclosure by the lawyer-client privilege or other applicable law. Answer choice D is incorrect because there is no blanket exception to the duty of confidentiality when a lawyer is called as a witness.

An attorney represented an actor in a contract dispute with the producer of a play. During the dispute, the attorney and the actor had a brief sexual relationship. Prior to engaging in sexual relations, the attorney informed the actor that the attorney was prohibited from having sex with a client. The actor dismissed the attorney's concerns, saying that it didn't matter. Sexual relations between the two terminated before the dispute was resolved. The dispute was resolved to the satisfaction of the actor. Was the attorney's action regarding the relationship proper? A. No, because an attorney may not engage in a sexual relationship with a client. B. No, because a client may not waive a conflict stemming from a sexual relationship with the attorney. C. Yes, because the client consented to the sexual relationship despite knowledge of the prohibition. D. Yes, because the contractual dispute was resolved to the satisfaction of the client after the sexual relationship with the attorney terminated.

Answer choice B is correct. An attorney is generally prohibited from engaging in sexual relations with a client. This conflict of interest may not be waived by the client through informed consent. Answer choice A is incorrect because an attorney may engage in a sexual relationship with a client if the sexual relationship existed before the attorney-client relationship began. Answer choice C is incorrect because a conflict arising from a sexual relationship may not be waived by informed consent. Answer choice D is incorrect because neither the termination of the sexual relationship before the termination of the representation nor the resolution of the matter to the satisfaction of the client excuses an attorney's sexual relationship with a client.

An attorney accepted a libel case, though he had no experience in that type of action and had little knowledge related to the issues involved. He informed his client of his inexperience but promised to work with a fellow competent attorney to handle the claim effectively. He received the client's written consent acknowledging this fact and allowing the attorney to proceed with the representation. The attorney then called his friend, a fellow attorney who had graduated at the top of her law school class but who likewise had no libel experience or knowledge herself, for advice on how to proceed. The friend provided good advice on general litigation techniques, and the attorney heeded this advice but took no other action. Would the attorney's representation of the client subject him to discipline? A. Yes, because he lacked the knowledge or experience to successfully represent the client. B. Yes, because he lacked the knowledge or experience to competently represent the client. C. No, because he consulted his friend on the matter. D. No, because the client consented in writing to the representation.

Answer choice B is correct. An attorney lacking the necessary knowledge or experience must: (i) decline or withdraw from representing the client, (ii) become competent without unreasonable delay, or (iii) associate with competent counsel. An attorney should not accept representation in a matter unless it can be performed competently. An attorney who represents a client incompetently is subject to discipline, regardless of causation or any financial loss to the client. Here, the attorney was not knowledgeable in the area of libel, and advice on general litigation tactics would not provide him with the knowledge he needed to competently represent his client in that area. Answer choice A is incorrect because an attorney does not have to win the case; he must merely be competent. Answer choice C is incorrect because the attorney's consultation with his friend would not constitute an association with competent counsel, as she had no knowledge or experience with libel cases either. Answer choice D is incorrect because a client cannot consent to incompetent representation.

A client approached an attorney to represent her in a products liability case because the attorney claimed to be a specialist in products liability actions. He told the client that the National Council on Products Liability had certified him as a specialist in products liability resolution, even though he had lost half of the 100 products liability actions he had taken. The client consented to representation in writing after this discussion. The attorney lost the case on a technicality that would have been known to any specialist in products liability, though a general practice attorney would not have recognized the issue. The entire case was lost based on this error. After demonstrating that she suffered damages, would the client be likely to recover in a civil liability action against the attorney? A. Yes, because the attorney lost her case. B. Yes, because the attorney deviated from the applicable standard of care for products liability attorneys. C. No, because the attorney did not deviate from the applicable standard of care of a general attorney. D. No, because the client consented to the representation in writing after full disclosure.

Answer choice B is correct. An attorney owes a duty of care to the client. The duty of care is generally the competence and diligence exercised by attorneys of similar experience under similar conditions. If an attorney represents to the client that he has specialized expertise, then the attorney will be held to the standard of care of a specialist. Here, the standard of a specialist would apply, and the attorney likely deviated from this standard. Answer choice A is incorrect because losing a case alone is not a basis for civil liability. What is relevant is the standard of care that the attorney exercised in representing the client. Answer choice C is incorrect because while the attorney may not have deviated from a general attorney's standard of care, he did deviate from the standard of care of a specialist. Answer choice D is incorrect because if the attorney was negligent, then the client's consent would not mitigate his liability.

An attorney worked at a law firm that encouraged its attorneys to provide at least 100 hours of pro bono service per year. The attorney typically surpassed this goal. Last year, however, the attorney was involved in a long, complex trial, and he did not provide any pro bono service. The attorney did not make any contributions to nonprofit legal organizations during the year. Would the attorney's failure to provide pro bono services or to contribute to nonprofit legal organizations subject him to discipline? A. No, because the Model Rules allow pro bono hours exceeding the 50-hour minimum to be carried forward to subsequent years. B. No, because the Model Rules indicate only that an attorney should aspire to perform 50 hours of pro bono services annually. C. Yes, because the Model Rules require an attorney to provide a minimum of 50 hours of pro bono legal services each year. D. Yes, because the Model Rules require an attorney to either render a minimum of 50 hours of pro bono legal services each year or make a monetary contribution to nonprofit legal organizations in lieu of such service.

Answer choice B is correct. The Model Rules of Professional Conduct set forth an aspirational goal for an attorney to accept representation of clients who are unable to pay, suggesting that attorneys offer at least 50 hours of pro bono legal services per year. This goal is not enforced through the disciplinary process. Accordingly, the attorney would not be subject to discipline. Answer choice A is incorrect because an attorney may not be disciplined for failure to perform pro bono service, and thus it is irrelevant whether hours can be carried forward. Answer choice C is incorrect because the guidelines regarding pro bono service in the Model Rules of Professional Conduct are voluntary, and thus the attorney would not be subject to discipline for failure to meet the suggested minimum. Answer choice D is incorrect because the Model Rules of Professional Conduct do not subject an attorney to discipline for the failure to either provide pro bono service or to make a monetary contribution to a nonprofit legal organization.

A sole practitioner who specialized in family law shared office space with a sole practitioner who specialized in criminal law. Since they frequently referred matters to one another, they decided to formalize this arrangement by entering into a reciprocal referral agreement under which they each agreed to refer matters that fit within their respective specialties to each other. They agreed that the arrangement would remain in effect for one year, at which point they would review it. The agreement did not prohibit either attorney from referring cases to other attorneys, and neither hesitated to recommend a different attorney whom they believed would better handle a particular case. When making referrals to each other, neither attorney informed clients of their agreement. Were the attorneys' actions regarding the referral agreement proper? A. No, because reciprocal referral agreements are prohibited by the Model Rules of Professional Conduct. B. No, because they did not inform clients of the arrangement. C. Yes, because the arrangement was not exclusive. D. Yes, because the arrangement did not provide for either attorney to be paid for referring clients.

Answer choice B is correct. Under certain circumstances, an attorney may enter into a reciprocal referral agreement whereby each attorney agrees to refer clients to the other. The agreements must not be exclusive or of an indefinite duration, and the client must be informed of the nature and existence of the agreement. In this case, the clients were not informed of the agreement, and thus the attorneys did not act properly. Answer choice A is incorrect because reciprocal referral agreements are permitted by the Model Rules of Professional Conduct. Answer choice C is incorrect because although the agreement was not exclusive, the attorneys acted improperly when they failed to notify the clients of the agreement. Answer choice D is incorrect because although the attorneys did not pay each other for the referrals, they acted improperly by failing to inform clients of the agreement.

A woman applied for a fellowship after completing her graduate degree. The woman asked a state trial court judge to provide a letter of recommendation for her application. In the judge's letter, which was written on his official judicial letterhead, the judge explained that he, as the woman's godfather, had known her for her entire life. He went on to describe her work ethic and explained that she had shown a strong commitment to the issues supported by the fellowship for years. Were the judge's actions in writing the letter of recommendation proper? A. No, because a judge may not provide a letter of recommendation on official letterhead. B. No, because a judge may serve as a reference only for former employees. C. Yes, because a judge may use official letterhead for a personal reference letter. D. Yes, because there are no restrictions on the use of official letterhead for reference letters.

Answer choice C is correct. A judge may provide a reference or recommendation for an individual based upon the judge's personal knowledge. The judge may use official letterhead for a reference letter if the judge indicates that the reference is personal and if there is no likelihood that the use of the letterhead would reasonably be perceived as an attempt to use the judge's official position to exert pressure. In this case, the judge indicated that the reference was personal, and there is no indication that the judge was trying to exert inappropriate pressure on the fellowship selection committee. Accordingly, the judge's actions were proper. Answer choice A is incorrect because a judge may write a letter of reference on official letterhead as long as the conditions described above are satisfied. Answer choice B is incorrect because a judge may write a personal reference letter for individuals who never worked for the judge. Answer choice D is incorrect because a reference letter written on official letterhead must satisfy the conditions described above.

An attorney who maintained a solo law practice entered into substantive discussions with a law firm about joining the firm. As part of these discussions, the attorney revealed the identities of her clients, their status as current or former clients, and the matters for which she represented them. The purpose of these revelations, which did not compromise the attorney-client privilege or otherwise prejudice the clients, was to detect conflicts of interest that would arise from the attorney joining the firm. The attorney did not seek the informed consent of her clients before making these revelations. Due to an inability to resolve a potential conflict of interest, the attorney did not join the law firm. Were the attorney's revelations proper? A. No, because the attorney did not seek the informed consent of her clients before making these revelations. B. No, because the attorney did not join the law firm. C. Yes, because the purpose of the revelations was to detect conflicts of interest that would arise from the attorney joining the law firm. D. Yes, because the duty of confidentiality does not apply to disclosures to another attorney of information relating to the representation of a client.

Answer choice C is correct. Although an attorney has a duty of confidentiality with respect to information relating to the representation of a client, there is an exception for such disclosures that are made to detect and resolve conflicts of interest that occur when a lawyer is engaged in substantive discussions about changing employment or there is substantive discussion about a change in the composition or ownership of a firm. Answer choice A is incorrect because an attorney seeking to associate with a firm may make limited disclosure of confidential information without first seeking the informed consent of her clients unless the revelations would compromise the attorney-client privilege or otherwise prejudice the clients. Because these limitations did not apply here, the attorney did not need the informed consent of her clients before making the disclosures. Answer choice B is incorrect because the "detection of conflicts of interest" exception to the duty of confidentiality applies even when the attorney does not change employment, provided the disclosures are not made until substantive discussions regarding the new relationship have occurred. Answer choice D is incorrect because there is no blanket exception to the duty of confidentiality for disclosures regarding a client made by an attorney to another attorney. In fact, most such disclosures would violate this duty.

An elderly client hired an attorney to amend his will to provide the client's nephew with a specific tract of heavily wooded property that he knew the nephew loved to use for hunting. The wooded property was one of many tracts of land on a vast stretch of property owned by the client. After the client died, the nephew discovered that the tract of land actually left to him was a patch of rocky shore abutting a large lake two plots down from the wooded property. The error occurred because the attorney had mistakenly listed the incorrect address for the property that the client sought to transfer to his nephew. The nephew, having been told by his grandfather that he would receive the wooded property, filed an action to have the wooded property transferred to him. During the estate proceedings, the court held that there was insufficient evidence to establish that the client intended to transfer the wooded property to the nephew. They instead awarded him the rocky land abutting the lake. The nephew filed a negligence action against the attorney. The attorney moved to dismiss the nephew's claim and argued that he owed no duty to the nephew.Would the nephew be entitled to file such a claim under a negligence theory? A. No, because the attorney did not owe a duty to the nephew. B. No, because the attorney did not represent the nephew. C. Yes, because the attorney knew that his client wanted the will to provide the nephew with the wooded property. D. Yes, because the attorney breached a duty to his client.

Answer choice C is correct. An attorney may owe a duty to a non-client when the attorney knows that the client intends the attorney's services to benefit the non-client. An attorney will owe a duty to a non-client to use care when and to the extent that: (i) the attorney knows that the client intends as one of the primary objectives of the representation that the attorney's services benefit the non-client, (ii) such duty would not significantly impair the attorney's performance of obligations to the client, and (iii) the absence of such duty would make enforcement of those obligations to the client unlikely. Here, the attorney knew that the client hired him to draft a will to benefit the nephew, and nothing about the arrangement seems to have impaired the attorney's ability to represent the client adequately. Answer choice A is incorrect because in this instance, the attorney would owe a duty to the nephew. Answer choice B is incorrect because attorneys may owe duties to parties other than their clients. As explained above, the attorney in this case did owe a duty to the nephew even though the nephew is not his client. Answer choice D is incorrect because the nephew, a third party, would not recover only because the attorney breached a duty to his client, the uncle. Rather, the nephew would be able to recover under a negligence theory in his own right, because the attorney owed a duty to the nephew as an intended beneficiary of the representation.

An attorney and her client met to discuss a discovery request from the opposing party. One item requested was a computer disk that contained evidence of the client's fraudulent behavior. The defendant told her attorney that if the attorney turned over the disk, the client would go "straight to jail." The attorney told the client that her words were protected by attorney-client privilege, and therefore, he would not reveal what she just stated. The attorney told the client to destroy the disk because the attorney was afraid that if he saw the information included on the disk, he might have to withdraw from representing her.Would the attorney's advice to his client subject him to discipline? A. No, unless the client actually destroyed the disk. B. No, because the attorney did not know if the disk included any relevant information. C. Yes, because the attorney counseled the client to destroy the disk. D. Yes, because the attorney did not report the information to the appropriate legal authority.

Answer choice C is correct. An attorney must not unlawfully obstruct another party's access to evidence or unlawfully alter, destroy, or conceal a document or other material having potential evidentiary value. An attorney must also not counsel another person to do or assist another person in doing any such act. This rule applies to evidentiary material generally, including computerized information. By telling the client to destroy the disk, which had clear evidentiary value, the attorney subjected himself to discipline. Answer choice A is incorrect because merely advising the client to destroy the disk would subject the attorney to discipline, regardless of whether the client followed the advice. Answer choice B is incorrect because the attorney was aware that the disk had evidentiary value, particularly in light of the client's comments. Answer choice D is incorrect because the attorney was not required to report this information.

An attorney represented a client who the attorney reasonably believed had a severe mental disorder affecting her ability to make rational personal and business decisions. Rather than consulting a physician or other mental healthcare professional about how to deal with his client, the attorney sought the advice of another attorney, who specialized in representing other attorneys in malpractice actions, about his professional obligations to his client. During his discussions with the other attorney, the attorney disclosed information that the client had revealed to him. Was the attorney's disclosure of information learned from his client proper? A. No, because the disclosed information was acquired during the course of the attorney's representation of the client. B. No, because the lawyer failed to consult a physician or mental healthcare professional first. C. Yes, because the lawyer may disclose this information to obtain advice regarding his professional obligations. D. Yes, because the lawyer for a client under a disability must disclose information necessary for protective action.

Answer choice C is correct. An attorney's duty of confidentiality does not prohibit an attorney from obtaining confidential legal advice about the attorney's responsibility to comply with the Model Rules of Professional Conduct. Generally, disclosing information to obtain such advice would be impliedly authorized for the attorney to carry out the representation. Even if not impliedly authorized, the Model Rules of Professional Conduct allow such disclosure. Answer choice A is incorrect because even if the information was acquired in the course of the representation, the attorney may still disclose the information to obtain confidential legal advice. Answer choice B is incorrect because the attorney does not have such an obligation before securing confidential legal advice on how to proceed in accordance with the Model Rules of Professional Conduct. Answer choice D is incorrect because the attorney does not have to reveal information about a client under a disability in order to take protective measures for that client. However, the Rules permit the attorney to make such disclosures to the extent reasonably necessary to protect the client's interests.

During a conversation among friends in his home, an attorney called into question whether a candidate for the office of attorney general met the statutory requirements. Specifically, the attorney stated that he had heard a rumor that the candidate had not been licensed to practice law in the state for the requisite number of years mandated by statute. When asked whether he had investigated this matter, the attorney replied that he didn't need to check the facts, it had to be true. The candidate had, in fact, been licensed to practice law for the statutorily required time period.Is the attorney subject to discipline? A. No, because the statement did not concern a current member of the judiciary or a candidate for judicial office. B. No, because the statement was not made in a public forum. C. Yes, because the false statement was made with a reckless disregard for its truth or falsity. D. Yes, because the statement concerned a candidate for public office.

Answer choice C is correct. False statements by a lawyer can unfairly undermine public confidence in the administration of justice. The prohibition on such statements applies to the qualifications or integrity of not only a judge or a candidate for judicial office, but also a public legal officer, such as an attorney general, or a candidate for such office. Consequently, a lawyer must not make a statement that the lawyer knows to be false or with reckless disregard as to its truth or falsity (i.e., a false statement made with actual malice) concerning the qualifications or integrity of a judge, adjudicatory officer, or public legal officer, or of a candidate for election or appointment to judicial or legal office. Because the attorney evidenced a reckless disregard for the truth or falsity of his statement regarding the failure of the attorney general candidate to meet the statutory requirements, the attorney is subject to discipline. Answer choice A is incorrect because, as noted with regard to answer choice C, the prohibition on a false statement made with actual malice about the qualifications or integrity of an individual applies not only to a judge or a candidate for judicial office, but also to a public legal officer, such as an attorney general, or a candidate for such office. Answer choice B is incorrect because the prohibition applies to false statements made with actual malice, regardless of whether the statements are made in private or in public. Answer choice D is incorrect because the prohibition on false statements does not apply to any candidate for a public office, but is limited to candidates for judicial or legal offices, such as judges, attorney generals, prosecuting attorneys, and public defenders.

An attorney admitted to the bar never practiced law, but instead took a job as a financial planner. After several years as a successful financial planner, one of the attorney's clients accused him of lying to her about the entities in which her funds had been invested. The attorney admitted that he had lied to the client but argued he had been acting in her best interest because she was overly risk averse. The attorney pointed out that the client had actually made more money for her than she would have made otherwise. The client filed a negligence action against the attorney and reported him to the local disciplinary commission. Would the attorney's actions as a financial planner subject him to discipline? A. No, because he was not engaged in the practice of law. B. No, because the client did not suffer any harm. C. Yes, because he engaged in conduct involving dishonesty. D. Yes, because the client filed a negligence action against the attorney.

Answer choice C is correct. It is professional misconduct for an attorney to engage in conduct involving dishonesty, fraud, deceit, or misrepresentation. An attorney need not be engaged in the practice of law to violate the Model Rules of Professional Conduct and be subject to discipline. Because the attorney engaged in conduct involving dishonesty, he would be subject to discipline. Answer choice A is incorrect because an attorney need not be engaged in the practice of law to be subject to discipline. Answer choice B is incorrect because an attorney is subject to discipline if he engages in conduct involving dishonesty, regardless of whether it results in actual harm. Answer choice D is incorrect because the mere fact that a person has filed a legal action against an attorney is not in and of itself evidence of professional misconduct.

A home builder employed an attorney to draft a contract of sale for a house the builder had constructed. The builder sold the house to a buyer who assumed a personally significant debt in order to purchase the house. After the sale, a friend of the builder's told the attorney that the builder, in violation of the law, had built the house on a landfill. The attorney contacted the builder and urged the builder to discuss the matter with the buyer. Although the builder admitted to the criminal act, the builder refused. The lawyer took no further action. Subsequently, although the landfill did not represent a health threat, the buyer had to sell the house for less than half of what he had paid for it.Is the attorney subject to discipline for failing to reveal the builder's criminal action to the buyer? A. Yes, because the client had used the attorney's services in furtherance of a crime. B. Yes, because the crime resulted in substantial injury to the financial interests of the buyer. C. No, because of the duty of confidentiality. D. No, because, at the time that the attorney learned of the crime, it had already occurred.

Answer choice C is correct. Subject to a few exceptions, a lawyer is prohibited from disclosing information relating to the representation of a client, whether that information was obtained from a third party or from the client, and whether it was acquired after the termination of the representation. While there is an exception for the disclosure of confidential information to prevent, mitigate, or rectify substantial financial harm to another based on a client's fraud or crime in furtherance of which the client has used the lawyer's services, the exception allows but does not require disclosure. Accordingly, the attorney is not subject to discipline for keeping the information confidential. Answer choice A is incorrect because, although the client had used the attorney's services in furtherance of a crime, the applicable exception that permits disclosure of confidential information in such circumstances does not require such disclosure. Answer choice B is incorrect because, although the crime resulted in substantial injury to the financial interests of the buyer, disclosure is not required. Answer choice D is incorrect because the exception to the disclosure of confidential information based on a client's fraud or crime encompasses situations in which the attorney learns of the crime after it has occurred. However, the exception does not require the attorney to disclose confidential information, but merely permits it.

A farmer asked his attorney to draft a letter in connection with a loan transaction. The letter issued by the attorney stated that the farm equipment to be pledged by the farmer to the lender to secure the loan was not subject to prior liens. The letter did not indicate that the attorney had relied on the farmer's statement to that effect and had not performed, as custom would require, a search of the relevant public records. Such a search would have revealed that the farmer's statement was false. At the farmer's direction, the attorney, unaware that the farmer had misinformed him as to the existence of the liens, sent the letter to the lender, who relied on it in making the loan. Subsequently, the farmer defaulted on the loan and, due to the prior liens, the lender was unable to recoup the outstanding balance owed by the farmer on the loan. Is the lawyer likely subject to civil liability to the lender? A. No, because the lender was not the attorney's client. B. No, because the attorney relied on his client's statement. C. Yes, because the attorney failed to comply with his duty of care with regard to the letter. D. Yes, because, by issuing the letter, the attorney became a guarantor of the loan.

Answer choice C is correct. While generally a lawyer does not owe a duty to a non-client, a lawyer can owe a duty of care to a non-client when the non-client has been invited to rely on the opinion or legal services of the lawyer, and the non-client does so. This is particularly true when the lawyer is aware that his opinion is being used with regard to a specific transaction and is being supplied to a particular person, rather than the public in general. In this instance, since the attorney failed to search publicly available records, as custom would require, he violated that duty of care. Moreover, the lender was harmed by the attorney's breach of this duty. Answer choice A is incorrect because, although generally a lawyer does not owe a duty to a non-client, a lawyer may owe a duty to a non-client when the non-client has been invited to rely on the opinion or legal services of the lawyer, and the non-client does so to her detriment. Answer choice B is incorrect because, although a lawyer who relies on information provided by a client is generally not liable to the client for doing so, such reliance may not protect the lawyer from liability to a third party when, in so relying, the lawyer fails to exercise reasonable care. Answer choice D is incorrect because the attorney, in issuing an opinion, did not assume an obligation with respect to the loan itself.

A client, during a conference with her attorney in his office about the client's pending divorce, threatened to kill her husband, from whom she was separated. Based on his knowledge of the client, the attorney believed his client's threat to be credible. The attorney took no action to warn the client's husband. Shortly thereafter, the client shot her husband, seriously wounding him.Is the attorney subject to discipline for his failure to warn his client's husband? A. Yes, because the attorney had a duty to warn his client's husband. B. Yes, because the attorney's failure to warn the client's husband was the cause of the husband's harm. C. No, because, since the client's husband did not die, the client did not accomplish her threat. D. No, because the attorney owed his client a duty of confidentiality.

Answer choice D is correct. A lawyer has an ethical duty of confidentiality to his client, which includes protecting confidential communications received from a client. MRPC 1.6(a). Since the communication regarding the client's intention to kill her husband was received by the attorney in the course of his representation of the client in a divorce, the attorney had a duty not to reveal the client's threat, even to the subject of the threat, the client's husband. While the Model Rules allow a lawyer to reveal confidential information concerning the representation of a client to the extent that the lawyer reasonably believes disclosure is necessary to prevent reasonably certain death or bodily harm, the Model Rules do not require disclosure. Accordingly, the attorney here would not be subject to discipline for failing to warn the client's husband. Answer choice A is incorrect because, while a lawyer may reveal confidential information received from a client concerning the representation to the extent that the lawyer reasonably believes disclosure is necessary to prevent reasonably certain death or bodily harm, a lawyer is not obligated to do so. MRPC 1.6(b)(1). Answer choice B is incorrect because, while the causal connection between the attorney's failure to warn the client's husband and the husband's harm would be relevant in a civil action that seeks to impose liability on the attorney, it is not relevant in a disciplinary action based on that failure. Answer choice C is incorrect because whether the client succeeded in accomplishing her threat is irrelevant to the issue of whether the attorney had a duty to warn the client's husband of that threat.

An attorney met with a client on a Monday morning about the client's pending divorce. The client told the attorney that if his wife did not agree to a property settlement by the end of the week, he "was going to kill [her]." The attorney thought his client was simply being dramatic but was concerned because he knew that his client already owned a gun and had made similar statements before. The attorney worried that the client might be serious about the threat and decided that, if the case had not settled by Friday, he would call the client's wife and warn her. The client shot his wife on Thursday. Is the attorney subject to discipline for not disclosing the client's threat to kill his wife? A. Yes, because the attorney reasonably believed that the client was going to harm his wife if the case did not settle. B. Yes, because the attorney owed a duty of fairness to the wife as the other party in the civil action. C. No, because the threat was not imminent since the client claimed he would kill his wife at the end of the week if a settlement was not reached. D. No, because the attorney had no obligation to disclose information he thought was necessary to prevent future bodily harm.

Answer choice D is correct. An attorney may reveal confidential information concerning the representation of a client to the extent the attorney reasonably believes disclosure is necessary to prevent reasonably certain death or bodily harm. The attorney is not obligated to do so, however. Here, the attorney thought his client was simply being dramatic, and the client had not taken steps to harm his wife despite making similar statements in the past. Answer choice A is incorrect because the fact that the belief was reasonable does not require the attorney to disclose the information: it simply permits him to do so. Answer choice B is incorrect because although an attorney does owe a duty of fairness to an opposing party, this duty does not require the attorney to disclose the information learned as a consequence of representing his client. Answer choice C is incorrect because the attorney was not obligated to disclose the information even if the threat was imminent. He would have the option of revealing the information, however.

Two friends were involved in a car accident and hired an attorney to represent them as co-parties in a claim for damages related to the accident. Each client provided the attorney with the minimum amount he would be willing to accept to settle the case. The clients suggested vastly different amounts. The attorney received a large settlement offer that met the individual requirements of each party, though each party would receive significantly different amounts in the settlement. He called both clients into his office separately to discuss settlement. Both clients immediately consented in writing to the agreement, though neither was aware of the amount received by the other. The attorney then accepted the offer on behalf of both of the clients. The clients met for dinner the following night to celebrate their win, and during dessert, one of the clients disclosed to the other the amount he had received in the settlement. The other client, who received a far smaller settlement, immediately called the attorney to express his dissatisfaction with the settlement. The attorney responded by saying that he accepted the amount his client had told him was acceptable.Would the attorney's actions regarding the settlement subject him to discipline? A. No, because he did not accept an amount below each client's specified amount. B. No, because both clients consented in writing to the settlement. C. Yes, because the parties received different amounts in the settlement. D. Yes, because the attorney did not disclose the extent of both settlement offers to both clients.

Answer choice D is correct. An attorney representing co-parties may make an aggregate settlement only if both clients consent in writing after full consultation and disclosure by the attorney, including disclosure of the nature and extent of all claims and pleas, and the participation of each party in the settlement. Here, the attorney did not disclose the nature and extent of the offer and the participation of each party in the settlement before accepting the settlement offer. Answer choice A is incorrect because the attorney did not make a full disclosure about the settlement to each party before he accepted the offers; it is irrelevant whether the amount was aligned with the original amount specified by each party. Answer choice B is incorrect because the attorney did not make a full disclosure regarding the settlement to the clients before they accepted the settlement in writing. Answer choice C is incorrect because an attorney may represent co-parties receiving different settlement amounts as long as the attorney makes a full disclosure to each client.

An attorney successfully represented a woman charged with operating a house of prostitution. Subsequently, a famous politician who was charged with soliciting a prostitute within the alleged brothel sought to hire the attorney to represent him. The woman told the attorney that she did not object to his representation of any of the men charged with solicitation, but she refused to sign a written statement to that effect because she no longer wanted to be linked to the charges in any written document. Would it be proper for the attorney to accept the politician as a client? A. No, because the attorney represented the woman previously. B. No, because the politician's claim is substantially related to the matter in which the attorney represented the woman. C. Yes, because the attorney received the woman's informed oral consent. D. Yes, although the attorney did not receive the woman's informed consent in writing.

Answer choice D is correct. An attorney who has previously represented a client in a matter must not subsequently represent another person in the same or a substantially related matter in which that person's interests are materially adverse to the interests of the former client, unless the former client gives informed consent, confirmed in writing. Here, while the matters were substantially related, the interests of the politician and the woman were not adverse because both wanted to disprove that any illegal acts related to prostitution occurred. Answer choice A is incorrect because even though the attorney represented a former client in a substantially related matter, the interests of the woman and the politician are not adverse. Answer choice B is incorrect because the former and current clients do not have adverse interests. Answer choice C is incorrect because the woman's informed consent, oral or written, is not required because the clients do not have adverse interests.

After working for several years in the civil division of a state attorney general's office, an attorney left the attorney general's office and joined a private law firm. The law firm represented a defendant in the appeal of his criminal conviction, which had been obtained by the criminal division of the attorney general's office while the attorney was employed in the civil division. The attorney was assigned to the team representing the defendant. The attorney did not seek the consent of the attorney general's office to the attorney's participation in the firm's representation of the defendant. Is the attorney's participation in the firm's representation of the defendant in an appellate action proper? A. No, because the attorney did not obtain the attorney general's consent prior to his representation of the defendant. B. No, because the defendant's conviction was obtained by the attorney general's office while the attorney worked there. C. Yes, because the attorney did not acquire confidential government information about the client while working at the attorney general's office. D. Yes, because the attorney did not participate in the defendant's conviction while working at the attorney general's office.

Answer choice D is correct. The Model Rules of Professional Conduct prohibit a former government attorney from representing a client in a matter in which the attorney participated personally and substantially as a government attorney, unless the appropriate government agency gives its informed written consent. In this case, the attorney worked on civil matters exclusively during his government tenure and did not work on the matter in question. Accordingly, his representation of the client would be proper even without informed consent. Answer choice A is incorrect because the attorney general's office did not need to consent, as the attorney did not work on the matter. Answer choice B is incorrect because although the matter was ongoing while the attorney worked in the attorney general's office, the attorney did not participate in the matter and thus would not be prohibited from representing the client. Answer choice C is incorrect. A government attorney who acquires confidential government information about a person may not later represent a private client whose interests are adverse to that person in a matter in which the information could be used to that person's material disadvantage. In this case, even assuming the attorney gained information about the defendant, the defendant was now the attorney's client.

A lawyer's act is considered to be that of the client in proceedings before a tribunal or in dealings with a third person if the tribunal or third person reasonably assumes that the lawyer is authorized to do the act on the basis of the client's (and not the lawyer's) manifestations of such authorization. Restatement (Third) of the Law Governing Lawyers § 27. Simply retaining a lawyer confers broad apparent authority on the lawyer unless other facts apparent to the third person show that the lawyer's authority is narrower. By retaining the lawyer, the client implies that the lawyer is authorized to act for the client in matters relating to the representation and reasonably appropriate under the circumstances to carry out the representation.

At the suggestion of a judge, the attorneys for both parties to a civil action agree to waive additional discovery and to begin the trial immediately. The judge does not know that the client of one of the attorneys has instructed its lawyers in writing not to bring cases to trial without specified discovery, some of which the attorney has not yet accomplished in the case. Although the attorney lacked actual authority to waive discovery here, the attorney had apparent authority, from the point of view of the judge, and the judge may hold the client to the immediate trial date. The client, however, may seek a discretionary release from the waiver from the judge, and may be able to obtain damages from the attorney for acting beyond the scope of her specific authority.

The principle of client-lawyer confidentiality is given effect by related areas of the law: (i)attorney-client privilege, (ii) the work product doctrine, and (iii) the ethical duty of confidentiality. Attorney-client privilege and work-product doctrine apply in judicial and other legal proceedings in which a lawyer may be called as a witness or otherwise required to produce evidence with regard to a client. The duty of client-lawyer confidentiality applies in situations other than those in which evidence is sought from the lawyer through legal compulsion. It applies not only to matters communicated in confidence by a client but also to all information relating to the representation of the client, from any source. A lawyer may not disclose such information except as authorized or required by the Model Rules or by other law. A court cannot order the revelation of material subject to the attorney-client privilege, but may order the disclosure of information protected by the duty of confidentiality that is not subject to that privilege.

Be certain to understand the difference between the duty of confidentiality and the evidentiary attorney-client privilege. Remember that only client-driven confidential communication is covered by the privilege. The duty of confidentiality also covers third party-driven communication, lawyer observation, and all information relating to the representation.

If the prohibition is based on a personal interest of the disqualified lawyer and does not present a significant risk of materially limiting the representation of the client by the remaining lawyers in the firm, the disqualified lawyer's conflict is not imputed to the firm. MRPC 1.10(a)(1). Disqualification arising from a close family relationship, for example, is considered personal and ordinarily is not imputed to members of the firm with whom the lawyer is associated. MRPC 1.7, cmt. 11.

If a lawyer is involved in a sexual relationship with the client, the Model Rules provide that the resulting conflict is personal to the lawyer and is not imputed to others in the lawyer's firm. MRPC 1.8(k)

Fee splitting among lawyers at different firms is permitted if all three of the following conditions are met: i) The fee is in proportion to the services rendered by each lawyer or joint responsibility is assumed for the representation; ii) The client agrees in writing to the fee splitting arrangement; and iii) The total fee charged must be reasonable (such that a client is not charged more just because additional lawyers are working on the case).

If all of these requirements are met, the lawyers can submit one bill to the client and then divide the fee.

A lawyer is prohibited from counseling or assisting the client in conduct the lawyer knows to be criminal or fraudulent. If the lawyer does advise or assist with a crime or fraud, the lawyer is subject to discipline as well as criminal or civil liability. A lawyer is permitted, however, to discuss the legal consequences of any proposed course of conduct with a client and may counsel or assist a client to make a good faith effort to determine the validity, scope, meaning, or application of the law. MRPC 1.2(d).

If the client's course of action has already begun and is continuing, the lawyer is required to avoid assisting the client and must withdraw from the representation. MRPC 1.2, cmt. 10; 1.16(a). In certain cases, withdrawal alone may be insufficient and the lawyer may have to give notice of the fact of withdrawal and publicly disaffirm any opinion, document, or affirmation previously provided to the client. MRPC 1.2, cmt. 10; 4.1

A law firm may be civilly liable for compensatory damages resulting from an injury caused by the wrongful conduct a principal (e.g., partner) or an employee of the firm who was acting in the ordinary course of the firm's business or with actual or apparent authority. Restatement (Third) of the Law Governing Lawyers § 58(1), cmt. f.

If the law firm is a general partnership, the partners are also jointly and severally liable with the firm. Restatement (Third) of the Law Governing Lawyers § 58(2), cmt. c. Individual partners need not be named in a suit against the partnership in order to collect a judgment out of partnership assets. However, a judgment against a partnership is not a judgment against its partners. Unless there is also a judgment against the partner, a judgment against a partnership cannot be satisfied from a partner's assets, only from the partnership's assets. Revised Uniform Partnership Act § 307(a-c), inc. cmts. 2, 3. Even though a partner is personally liable for a partnership obligation, a partnership creditor generally must exhaust the partnership's assets before levying on the partners' personal assets. Exceptions exist when the partnership is a debtor in bankruptcy, the partner has consented, or the partner is liable independent of the partnership, such as when the partner was the primary tortfeasor. In addition, a court may authorize execution against a partner's assets when the partnership's assets are clearly insufficient, exhaustion of the partnership's assets would be excessively burdensome, or it is otherwise equitable to do so. Revised Uniform Partnership Act § 307(d); Restatement (Third) of the Law Governing Lawyers § 58(2), cmt. g. If the firm is a professional corporation, a limited liability partnership, or a limited liability company, a principal of the firm is typically not subject to vicarious liability, but can, of course, be liable for the principal's own negligence, which can include negligent supervision of another member or employee of the firm. A lawyer of a corporate law department or a governmental agency is not vicariously liable for damages cause by another lawyer in the department or agency. Restatement (Third) of the Law Governing Lawyers § 58(3), cmt. c.

Keep in mind that, unlike a successful malpractice action, violation of a rule of professional responsibility does not require a finding that a client has been harmed. MRPC, Scope, cmt. 20. A lawyer may be liable for malpractice based on negligence if the client establishes the elements of negligence, unless the lawyer has a valid defense. Restatement (Third) of the Law Governing Lawyers §§ 48-54.

In addition to professional discipline, a lawyer who violates an ethics rule may also potentially be subject to civil liability for malpractice. While sanctions for violation of the disciplinary rules are aimed at punishing a lawyer and protecting the public from future violations, damages for malpractice compensate an injured client or third party. A malpractice action is brought in a civil court and is prosecuted by the injured client or third party. The mere violation of a rule of professional responsibility does not automatically result in a finding of malpractice. It is not negligence per se. Violation of a rule is, however, generally treated as evidence that the lawyer's conduct violated the duty of care.

A lawyer may accept representation when the requisite level of competence can be achieved by reasonable preparation. This also applies to a lawyer who is appointed as counsel for an unrepresented person. MRPC 1.1, cmt. 4. Competent representation can also be provided through the association with a lawyer of established competence in the field. MRPC 1.1, cmt. 2.

In an emergency, a lawyer may give advice or assistance in a matter in which the lawyer does not have the skill ordinarily required if referral to or consultation or association with another lawyer would be impractical. Assistance should be limited, however, to what is reasonably necessary under the circumstances. MRPC 1.1, cmt. 3. Whether a lawyer is properly prepared is determined by the degree of complexity and consequence of the matter. Major litigation and complex transactions ordinarily require more extensive treatment than matters of lesser complexity and consequence. A lawyer's responsibility to prepare may further be limited by an agreement between the lawyer and client as to the scope of the representation. MRPC 1.1, cmt. 5.

Information that is imparted to a lawyer during and related to the representation of a client is attributed to the client for purposes of determining the client's rights and liabilities with regard to matters in which the lawyer represents the client, unless such rights or liabilities require proof of the client's personal knowledge or intentions or the lawyer's legal duties preclude disclosure of such information to the client. Restatement (Third) of the Law Governing Lawyers § 28(1). 1) Exception—transactions not within scope of representation A client is not charged with a lawyer's knowledge with regard to a transaction in which the lawyer does not represent the client. Restatement (Third) of the Law Governing Lawyers § 28, cmt. b. 2) Exception—other lawyers within the firm The knowledge of a lawyer not personally engaged in representing a client, but in the same firm, is not attributed to the client unless the lawyer acquiring the knowledge is aware that the information is relevant to the firm's representation of the client. Restatement (Third) of the Law Governing Lawyers § 28, cmt. b. 3) Exception—criminal liability A lawyer's knowledge is not attributed to the client to establish the client's criminal liability, but evidence of the lawyer's knowledge might be admissible to show what the client knew. Restatement (Third) of the Law Governing Lawyers § 28, cmt. b.

In the absence of applicable law otherwise, a third person may give notification to a client in a matter in which the client is represented by a lawyer by giving notification to the client's lawyer, unless the third person knows of circumstances reasonably indicating that the lawyer's authority to receive notification has been abrogated. Restatement (Third) of the Law Governing Lawyers § 28(2).

When the lawyer reasonably believes that the client has diminished capacity, is at risk of substantial physical, financial, or other harm unless action is taken, and cannot adequately act in her own interest, the lawyer may take reasonably necessary protective action, including consulting with individuals or entities that have the ability to take action to protect the client and, in appropriate cases, seeking the appointment of a guardian ad litem, conservator, or guardian to act on behalf of the client. MRPC 1.14(b).

Information that relates to the representation of a client with diminished capacity is protected by the confidentiality provisions of MRPC 1.6. Thus, generally, unless authorized to do so, the lawyer may not disclose such information. When taking protective action, including seeking the appointment of a guardian, however, the lawyer is impliedly authorized to reveal information about the client, even if the client directs otherwise, but only to the extent reasonably necessary to protect the client's interests. MRPC 1.14(c). The lawyer should determine whether it is likely that the person or entity consulted with will act adversely to the client's interests before discussing matters related to the client. MRPC 1.14, cmt. 8.

The client-lawyer relationship begins when the client reasonably believes the relationship exists. No formal writing or agreement is required. No payment of a fee is necessary.

On the exam, look for casual conversations between a lawyer and a potential client. If the facts indicate the client reasonably believes the relationship exists, then it does.

A lawyer is prohibited from making an agreement prospectively limiting malpractice liability to a client, unless the client is represented by an independent lawyer in making the agreement. MRPC 1.8(h)(1).

Merely advising a client to seek the counsel of an independent lawyer is not sufficient to satisfy this exception to the prohibition on an agreement to limit a malpractice recovery. The client must be represented by such a lawyer in making the agreement. Also note that this provision does not require that the agreement be in writing or that the client sign the agreement. An agreement to submit a future malpractice dispute to arbitration is permitted when the client is informed as to the scope and the effect of the arbitration clause. MRPC 1.8(h), cmt. 14.

A lawyer may not solicit a substantial gift or prepare an instrument (such as a will, trust agreement, or deed) that gives a substantial gift to the lawyer or a person related to the lawyer, unless the client is related to the donee. Related persons include a spouse, child, grandchild, parent, grandparent, or other relative or individual with whom the lawyer or the client maintains a close, familial relationship. MRPC 1.8(c), cmts. 6, 7

Note that a lawyer may accept an unsolicited gift from a client, if the transaction meets general standards of fairness. For example, a simple gift such as a present given at a holiday or as a token of appreciation is permitted. If a client offers the lawyer a more substantial gift, the rule does not prohibit the lawyer from accepting it, but such a gift may be voidable by the client under the doctrine of undue influence, which treats client gifts as presumptively fraudulent. MRPC 1.8(c), cmt. 6.

A lawyer does have a duty to accept court appointments, so long as no good cause exists to decline. MRPC 6.2. Good cause to decline representation exists if: (i) the lawyer could not handle the matter competently; (ii) if the representation would result in an improper conflict of interest (as for example, when the client or the cause is so repugnant to the lawyer as to be likely to impair the client-lawyer relationship or the lawyer's ability to represent the client); or (iii) if acceptance would be unreasonably burdensome, for example, when it would impose a financial sacrifice so great as to be unjust. MRPC 6.2, cmt. 2.

Note that an appointed lawyer has the same obligations to the client as retained counsel, including the obligations of loyalty and confidentiality, and is subject to the same limitations on the client-lawyer relationship, such as the obligation to refrain from assisting the client in violation of the Model Rules.

If a lawyer switches firms and has a conflict of interest because of the lawyer's previous representation of a client or the prior law firm's previous representation of a client pursuant to MRPC 1.9, the disqualified lawyer's conflict is not imputed to the new law firm if: i) The disqualified lawyer is timely screened from any participation in the matter and is apportioned no part of the fee; ii) Written notice is promptly given to any affected former client so that the client can determine if there has been compliance with the Model Rules; and iii) Certifications of compliance with the Model Rules and with the screening procedures are provided to the former client by the screened lawyer and by a partner at the firm at reasonable intervals, upon written request by the former client, and if the screening procedures are terminated. MRPC 1.10(a)(2).

Note that if all of the above requirements are met, the former client's consent is not required. The notice provided to the affected former client must describe the screening procedures used, state compliance with the confidentiality provisions of the Model Rules by the disqualified lawyer and the firm, indicate that review before a tribunal may be available, and provide that the firm will respond promptly to any written questions or objections by the client about the screening procedures. MRPC 1.10(a)(2)(ii). The requirement that the disqualified lawyer not be apportioned any part of the fee does not prohibit the lawyer from receiving a salary or partnership share that is established by prior independent agreement. The disqualified lawyer may not receive compensation that is directly related to the matter from which the lawyer is disqualified. MRPC 1.10, cmt. 8.

Under the Sarbanes-Oxley Act (SOX), 17 CFR §§ 205.1-205.7, the Securities and Exchange Commission (SEC) established rules of professional conduct for securities lawyers. Securities lawyers practice before the SEC and represent an issuer of securities (publicly-traded corporation). SOX § 307 obligates securities lawyers to report any evidence of reasonably likely material violations of federal or state securities laws to the chief legal officer or both the chief legal officer and chief executive officer within the corporation.

Note that the SEC rule requires that the violation merely be "reasonably likely," whereas MRPC 1.13 requires actual knowledge of a material violation. A securities lawyer may report a material violation directly to the SEC without the consent of the client in order to: (i) prevent the client from committing a violation that will cause substantial injury to the corporation or its shareholders; (ii) prevent the client from committing or suborning perjury; or (iii) to mitigate or remedy a financial injury to the client or shareholders. e) Compliance with or failure to comply with SOX A securities lawyer cannot be subject to professional discipline or civil liability for reporting to the SEC or complying with the terms of SOX. A securities lawyer who does not comply with the terms of SOX is subject to civil penalties and may be denied the privilege of practicing before the SEC.

The Model Rules prohibit a former government lawyer from representing a client in a matter in which the lawyer participated personally and substantially as a government lawyer, unless the appropriate government agency gives its informed consent, confirmed in writing, to the representation. MRPC 1.11(a).

Note that the prohibition applies regardless of whether the lawyer is adverse to the government in attempting to represent the new client. Thus, a lawyer who has pursued a claim on behalf of the government may not pursue the same claim on behalf of a subsequent private client after the lawyer has left government service, except when the government agency gives its informed, written consent. If a former government lawyer is disqualified from representation, no lawyer in the firm with which the former government lawyer is associated may knowingly undertake or continue representation in the matter unless: (i) the disqualified lawyer is timely screened from any participation in the matter and gets no part of any fee from the matter; and (ii) written notice is promptly given to the appropriate government agency to enable it to ascertain whether the lawyer and firm are in compliance with the conflict rules. MRPC 1.11(b).

In general, a lawyer may not represent a client with interests materially adverse to those of a prospective client in the same or a substantially related matter if the lawyer received information from the prospective client that could be significantly harmful to that person in the matter. MRPC 1.18(c).

Representation is permissible if the affected and prospective clients each give informed consent, confirmed in writing. MRPC 1.18(d). Representation is also permissible if: (i) the lawyer who received the information took reasonable measures to avoid exposure to more disqualifying information than was reasonably necessary to determine whether to represent the prospective client; (ii) the disqualified lawyer is timely screened from any participation in the matter and gets no part of the fee; and (iii) written notice is promptly given to the prospective client.

Lawyers may provide "law-related services," which are defined by the Model Rules as services that might reasonably be performed in conjunction with and in substance are related to the provision of legal services, and that are not prohibited as unauthorized practice of law when provided by a non-lawyer. MRPC 5.7(b).

Some examples of law-related services include providing title insurance, financial planning, accounting, trust services, real estate counseling, legislative lobbying, economic analysis, social work, psychological counseling, tax preparation, and patent, medical, or environmental consulting. MRPC 5.7, cmt. 9. Under certain circumstances, the lawyer providing law-related services is subject to the Model Rules with regard to such services, even though a non-lawyer performing such services would not be subject to the rules.

A confidential communication between a client and her lawyer is privileged if it is made for the purpose of obtaining or providing legal assistance for the client. The evidentiary attorney-client privilege covers the client's communication to a lawyer whom the client reasonably believes represents the client, and only when the circumstances indicate a desire by the client for confidentiality. This privilege applies in judicial and other proceedings in which a lawyer may be called as a witness or otherwise required to produce evidence concerning a client. MRPC 1.6, cmt. c.

The communication must be intended to be confidential in order to be privileged. A communication made in the presence of a third party generally is not privileged, but a communication that is made in the reasonable belief that it is being made in confidence that is overheard by a third party is privileged. In addition, the presence of, or communication by or through, a representative of the client or the lawyer does not destroy the attorney-client privilege. 1) Waiver and disclosure A client may waive the privilege directly or by disclosure of the information communicated. The Federal Rules of Evidence address the effect that a litigation-related disclosure of protected information has on the waiver of the attorney-client privilege, drawing a distinction between an intentional disclosure and an unintentional disclosure. The rule applies to confidential communications as well as material protected by the work-product doctrine. Fed. R. Evid. 502. a) Inadvertent disclosure—no waiver When made during a federal proceeding, the inadvertent disclosure of privileged communication or information does not waive the privilege if the holder of the privilege: i) Took reasonable steps to prevent disclosure; and ii) Promptly took reasonable steps to rectify the error. Fed. R. Evid. 502(b). In determining whether the holder took reasonable steps to prevent disclosure, factors such as the number of documents to be reviewed, the time constraints for production, or the existence of an efficient records-management system may be relevant. b) Intentional disclosure—limitation on the scope of waiver When made during a federal proceeding, the intentional disclosure of privileged material operates as a waiver of the attorney-client privilege. The waiver extends to undisclosed information only in those unusual situations in which (i) the disclosed and undisclosed material concern the same subject matter and (ii) fairness requires the disclosure of related information because a party has disclosed information in a selective, misleading, and unfair manner. Fed. R. Evid. 502(a). c) Effect of disclosure made in a state proceeding When privileged material is disclosed in a state proceeding and the state and federal laws are in conflict as to the effect of the disclosure, the disclosure does not operate as a waiver in a subsequent federal proceeding if the disclosure (i) would not be a waiver had it been made in a federal proceeding or (ii) is not a disclosure under the law of the state where it was made. In other words, the federal court must apply the law that is most protective of the privilege. This rule does not apply if the state court has issued an order concerning the effect of the disclosure; in such a case, the state-court order would be controlling. Fed. R. Evid. 502(c). d) Controlling effect of a federal confidentiality order A federal court may order that the privilege or protection is not waived by disclosure connected with the pending litigation (i.e., a confidentiality order). In such a case, the disclosure does not constitute a waiver in any other federal or state proceeding. Fed. R. Evid. 502(d). e) Parties' agreement An agreement between the parties regarding the effect of a disclosure binds only the parties unless the agreement is incorporated into a court order. Fed. R. Evid. 502(e).

A lawyer may reveal confidential information concerning the representation of a client to the extent the lawyer reasonably believes disclosure is necessary to prevent reasonably certain death or substantial bodily harm. MRPC 1.6(b)(1).

Substantial bodily harm or death is reasonably certain to occur if it will be suffered imminently or if there is a present and substantial threat that a person will suffer such harm or death at a later date if the lawyer fails to take action necessary to eliminate the threat. MRPC 1.6, cmt. 6. Harm or death need not be imminent, but merely reasonably certain to occur if the lawyer does not disclose. This exception applies even if a third party—not the client—is the cause of the potential substantial bodily harm or death. This exception applies even if the substantial bodily harm or death would result from an unintentional act or natural causes. An attorney has been hired by a client to represent the client in a civil commitment proceeding. The client tells the attorney that she intends to commit suicide the next day, and the attorney believes that the client will carry out this threat. It would be proper for the attorney to disclose the client's intentions to the proper authorities, because there is a present and substantial threat that the client will suffer harm or death if the attorney fails to take action necessary to eliminate the threat. MRPC 1.6, cmt. 6.

In general, if one lawyer in a firm is prohibited by the conflict of interest rules from representing a client, that prohibition applies to all other lawyers in the firm.

Such a disqualification may be waived by the affected client if: (i) the lawyer reasonably believes that she will be able to provide competent and diligent representation to the affected client; (ii) the representation is not prohibited by law; (iii) the representation does not involve the assertion of a claim by one client against another client represented by the lawyer in the same litigation or other proceeding before a tribunal; and (iv) the affected client gives informed consent, confirmed in writing. MRPC 1.10(c); 1.7. The rule of imputed disqualification does not prohibit representation by others in a law firm when the person prohibited from involvement in a matter is a non-lawyer (e.g., paralegal, legal secretary). Such persons, however, must generally be screened from any personal participation in the matter to avoid communication to others in the firm of confidential information that both the non-lawyers and the firm have a legal duty to protect. If a nonlawyer employee in fact conveys confidential information learned about a client in one firm to lawyers in another, a prohibition on representation by the second firm is warranted. (Note that a similar approach is required when a lawyer is prohibited from involvement in a matter because of events before the individual became a lawyer, such as involvement in a matter as a law student.) MRPC 1.10, cmt. 4; Restatement (Third) of the Law Governing Lawyers § 123, cmt. f.

A lawyer with direct supervisory authority over the work of another lawyer in the firm or a non-lawyer employed by the firm must make reasonable efforts to ensure that the supervised person's conduct conforms to the Model Rules. MRPC 5.1(b).

Supervisory obligations are frequently tested in situations in which non-lawyers are not properly instructed of the need to maintain confidential communications.

A lawyer is subject to the Model Rules with respect to the provision of law-related services by an entity controlled by the lawyer individually or with others if the lawyer fails to take reasonable measures to assure that a person obtaining the law-related services knows that the services are not legal services and that the protections of the client-lawyer relationship do not exist. MRPC 5.7(a)(2).

The burden is on the lawyer to show that the lawyer has taken reasonable measures under the circumstances to communicate the desired understanding. Thus, a sophisticated user of law-related services, such as a publicly-held corporation, may require a lesser explanation than someone unaccustomed to making distinctions between legal services and law-related services. MRPC 5.7, cmt. 7.

A lawyer who has managerial authority over the professional work of a law firm, such as a partner of a partnership or the head of a corporation's legal department, or a lawyer with intermediate managerial responsibilities, such as the head of the litigation branch of a law firm, must make reasonable efforts to ensure the firm has measures in place that give reasonable assurance that the conduct of all lawyers in the firm, as well as that of non-lawyers employed by, retained by, or associated with the firm, conforms to the Model Rules. MRPC 5.1(a), 5.3(a). What constitutes reasonable measures depends on the size and kind of firm.

The obligations of a partner are tested frequently in the context of paralegals or other non-lawyers violating a Model Rule without the lawyer's knowledge. The outcome depends on whether the firm had procedures in place to prevent such violations. If a question indicates that a subordinate engaged in misconduct without the lawyer's knowledge, check to see if preventative measures, such as employee training, were taken.

A lawyer with managerial authority in a law firm or a lawyer with direct supervisory authority over another lawyer or non-lawyer employed by the firm is subject to discipline for misconduct about which the managerial or supervisory lawyer knows at a time when the consequences of the misconduct can be avoided or mitigated, but the managerial or supervisory lawyer fails to take reasonable remedial action. MRPC 5.1(c); 5.3(c).

This is a frequently tested area on the MPRE. Often the issue will involve whether the supervisor could have prevented the misconduct. If the supervisor could not have prevented the subordinate's misconduct, then the subordinate's misconduct is not attributed to the supervisor unless the supervisor learned of the action while it was still possible to take remedial action to mitigate the consequences of the misconduct.

A lawyer must not obtain a proprietary interest in the cause of action or subject matter of litigation in which a client is represented, except when: (i) the lawyer acquires a lien granted by law to secure payment of a fee; or (ii) the lawyer contracts for a reasonable contingent fee, provided the case is not a criminal or matrimonial/domestic matter. MRPC 1.8(i).

When a lawyer acquires by contract a security interest in property other than that recovered through the lawyer's efforts in the litigation, such an acquisition is a business or financial transaction with a client and is subject to the requirements of MRPC 1.8(a) (see V.D.1. Business transactions with client, infra). MRPC 1.8(i), cmt. 16.

Model Rules require that a person's informed consent be "confirmed in writing," the informed consent need not take the form of a document signed by the person. In such circumstances, the consent can be given orally and a writing can be prepared by the lawyer as a confirmation of the oral consent. If it is not possible to obtain or transmit the writing at the time the informed consent is given, the lawyer must obtain and transmit it within a reasonable time thereafter, but may immediately act in reliance on the consent. MRPC 1.0(b), cmt. 1.

When the phrase "reasonably should know" is used, a lawyer of reasonable prudence and competence would ascertain the matter in question (an objective standard). MRPC 1.0(a),(f),(h),(i),(j).

A "law firm" (or "firm") is any association of lawyers authorized to practice law, and includes a partnership or professional corporation as well as lawyers employed in a legal services organization or the legal department of a corporation or other organization. MRPC 1.0(c).

Whether two or more lawyers constitute a firm depends on the specific facts such as: - how they hold themselves out to the public - the terms of any formal agreement - access to information about clients are relevant The mere sharing of office space by lawyers coupled with occasional consultation or assistance is ordinarily insufficient to result in the lawyers being treated as a firm. In addition, the determination of whether two or more lawyers are a firm for purposes of one rule is not necessarily conclusive with respect to another rule. For example, a lawyer whose representation of a party would violate the conflict-of-interest rule because of another lawyer's representation of the opposing party might not be subject to the rule that imputes to the lawyer information known by the other lawyer. MRPC 1.0, cmt. 2. A lawyer practicing law as a sole proprietorship also falls within the definition of a "law firm." MRPC 1.0(c).

The attorney-client privilege does not protect these confidential communications:

i) Communications made to enable or aid the commission of what the client knew or should have known was a crime or fraud; ii) Communications relevant to a dispute between lawyer and client or former client (e.g., client's malpractice allegation, lawyer's compensation or reimbursement claim); iii) Communications relevant to a dispute between parties who claim through the same deceased client; and iv) Communications between former co-clients who are now adverse to each other.


Conjuntos de estudio relacionados

ISYS438 Project Risk Management Chapter 11 Terms

View Set

Ethical Hacking - C701 TotalTester Part 1/2

View Set

Unit # 1 : DNA + Protein Synthesis

View Set

Chapter 15 sports psych (Team cohesion in sport)

View Set

Chapter 4- Age of Religious Wars Nolan's Practice

View Set

World History AP 2013 Released Exam

View Set

Chapter 3: Providing equal employment opportunity and a safe workplace

View Set